Milestone: 100 day streak! (I just want to post a problem)

Geometry Level 5

x 3 24 x 2 + 180 x 420 = 0 x^3-24x^2+180x-420=0

A A B C \triangle ABC has sides a , b a,b and c c . It is known that a , b a,b and c c are also the roots of the equation above.

Given that cos A + cos B + cos C = p q \cos A+\cos B+\cos C =\dfrac{p}{q} , where p p and q q are coprime positive integers , find the value of 11 ( p + q ) + 25 11(p+q)+25 .


Inspiration .


The answer is 839.

This section requires Javascript.
You are seeing this because something didn't load right. We suggest you, (a) try refreshing the page, (b) enabling javascript if it is disabled on your browser and, finally, (c) loading the non-javascript version of this page . We're sorry about the hassle.

2 solutions

Rishabh Jain
Jul 12, 2016

cos A = b 2 + c 2 a 2 2 b c = ( b 2 + c 2 a 2 2 b c + 1 ) 1 \cos A=\dfrac{b^2+c^2-a^2}{2bc}=\left(\dfrac{b^2+c^2-a^2}{2bc}+\color{#D61F06}{1}\right)-\color{#D61F06}{1} Now manipulating the first bracket gives:

cos A = ( ( b + c ) 2 a 2 2 b c ) 1 = ( ( b + c a ) ( b + c + a ) 2 b c ) 1 \cos A=\left(\dfrac{(b+c)^2-a^2}{2bc}\right)-\color{#D61F06}{1}=\left(\dfrac{(b+c-a)(b+c+a)}{2bc}\right)-\color{#D61F06}{1}

Now by Vieta's formula: cyc a = 24 , cyc a b = 180 , a b c = 420 \color{teal}{\small{\displaystyle\sum_{\text{cyc}}a=24,\displaystyle\sum_{\text{cyc}}ab=180,abc=420}} so that:

cos A = ( 24 2 a ) 24 2 420 a 1 = 2 35 ( 12 a a 2 ) 1 \cos A=\dfrac{(24-2a)\cdot 24}{2\cdot\frac{420}a}-\color{#D61F06}{1}=\dfrac{2}{35}\left(12a-a^2\right)-\color{#D61F06}{1}

Now call the required expression K \mathcal K ,

K = cyc cos A = cyc [ 2 35 ( 12 a a 2 ) 1 ] = 2 35 [ 12 24 ( ( 24 ) 2 2 ( 180 ) ) ] 3 = 39 35 \begin{aligned}\mathcal K=\displaystyle\sum_{\text{cyc}}\cos A=&\displaystyle\sum_{\text{cyc}}\left[\dfrac{2}{35}(12a-a^2)-\color{#D61F06}{1}\right]\\=&\dfrac{2}{35}[12\cdot 24-((24)^2-2(180))]-\color{#D61F06}{3}\\=&\dfrac{39}{35}\end{aligned}

11 ( 39 + 35 ) + 25 = 839 \Large \therefore 11(39+35)+25=\boxed{\color{#007fff}{839}}

You always come up with something interesting...

Anyway, typo: When you calculate the sum, 1 \color{#D61F06}{-1} should not be inside there

Hung Woei Neoh - 4 years, 11 months ago

Log in to reply

Why... That 1 \color{#D61F06}{-1} is significant!

Rishabh Jain - 4 years, 11 months ago

Log in to reply

It's not multiplied by 15 86 \dfrac{15}{86} . I'm getting 270 43 -\dfrac{270}{43} from your solution

Hung Woei Neoh - 4 years, 11 months ago

Log in to reply

@Hung Woei Neoh Oh sorry ... That bracket was misplaced.

Rishabh Jain - 4 years, 11 months ago

Log in to reply

@Rishabh Jain Same thing for the second row

Hung Woei Neoh - 4 years, 11 months ago

Log in to reply

@Hung Woei Neoh I was correcting that only next, which I noticed after writing the reply.

Rishabh Jain - 4 years, 11 months ago

Typo: 2 ( 180 ) -2(\color{#D61F06}{180}) in the second last line

Sorry for causing you so much trouble.

And sorry to everyone who got confused by the question

Hung Woei Neoh - 4 years, 11 months ago

Log in to reply

No I'm still editing..

Rishabh Jain - 4 years, 11 months ago

Log in to reply

Nope, you're done. I already read your solution, you've edited everything already.

Again, sorry for all the trouble I caused

Hung Woei Neoh - 4 years, 11 months ago

Log in to reply

@Hung Woei Neoh I was typing done .. :-) No its normal. Nothing to be sorry for.

Rishabh Jain - 4 years, 11 months ago
Hung Woei Neoh
Jul 12, 2016

x 3 24 x 2 + 180 x 420 = 0 x^3-24x^2+180x-420=0

From Vieta's formula , we know that

a + b + c = 24 a b + a c + b c = 180 a b c = 420 \color{#3D99F6}{a+b+c=24}\\ \color{#D61F06}{ab+ac+bc=180}\\ \color{#EC7300}{abc=420}

From cosine rule , we also know that

a 2 = b 2 + c 2 2 b c cos A cos A = b 2 + c 2 a 2 2 b c a^2=b^2+c^2-2bc \cos A\\ \implies \cos A = \dfrac{b^2+c^2-a^2}{2bc}

Similarly, we can find that

cos B = a 2 + c 2 b 2 2 a c cos C = a 2 + b 2 c 2 2 a b \cos B = \dfrac{a^2+c^2-b^2}{2ac}\\ \cos C = \dfrac{a^2+b^2-c^2}{2ab}

Therefore,

cos A + cos B + cos C = b 2 + c 2 a 2 2 b c + a 2 + c 2 b 2 2 a c + a 2 + b 2 c 2 2 a b = a b 2 + a c 2 a 3 + a 2 b + b c 2 b 3 + a 2 c + b 2 c c 3 2 a b c = a b ( a + b ) + a c ( a + c ) + b c ( b + c ) ( a 3 + b 3 + c 3 ) 2 a b c = a b ( 24 c ) + a c ( 24 b ) + b c ( 24 a ) ( a 3 + b 3 + c 3 ) 2 a b c = 24 ( a b + a c + b c ) 3 a b c ( a 3 + b 3 + c 3 ) 2 a b c \cos A + \cos B + \cos C\\ = \dfrac{b^2+c^2-a^2}{2bc}+ \dfrac{a^2+c^2-b^2}{2ac}+ \dfrac{a^2+b^2-c^2}{2ab}\\ =\dfrac{ab^2+ac^2-a^3+a^2b+bc^2-b^3+a^2c+b^2c-c^3}{2\color{#EC7300}{abc}}\\ =\dfrac{ab(\color{#3D99F6}{a+b})+ac(\color{#3D99F6}{a+c})+bc(\color{#3D99F6}{b+c})-(\color{#69047E}{a^3+b^3+c^3})}{2\color{#EC7300}{abc}}\\ =\dfrac{ab(\color{#3D99F6}{24-c})+ac(\color{#3D99F6}{24-b})+bc(\color{#3D99F6}{24-a})-(\color{#69047E}{a^3+b^3+c^3})}{2\color{#EC7300}{abc}}\\ =\dfrac{24(\color{#D61F06}{ab+ac+bc})-3\color{#EC7300}{abc}-(\color{#69047E}{a^3+b^3+c^3})}{2\color{#EC7300}{abc}}

Now, we use Newton's sums to get a 3 + b 3 + c 3 \color{#69047E}{a^3+b^3+c^3} :

a 2 + b 2 + c 2 = ( a + b + c ) 2 2 ( a b + a c + b c ) = ( 24 ) 2 2 ( 180 ) = 216 a 3 + b 3 + c 3 = ( a + b + c ) ( a 2 + b 2 + c 2 ) ( a b + a c + b c ) ( a + b + c ) + 3 a b c = ( 24 ) ( 216 ) ( 180 ) ( 24 ) + 3 ( 420 ) = 2124 \color{#20A900}{a^2+b^2+c^2}=(\color{#3D99F6}{a+b+c})^2-2(\color{#D61F06}{ab+ac+bc})=(\color{#3D99F6}{24})^2-2(\color{#D61F06}{180})=\color{#20A900}{216}\\ \color{#69047E}{a^3+b^3+c^3} = (\color{#3D99F6}{a+b+c})(\color{#20A900}{a^2+b^2+c^2})-(\color{#D61F06}{ab+ac+bc})(\color{#3D99F6}{a+b+c})+3\color{#EC7300}{abc}= (\color{#3D99F6}{24})(\color{#20A900}{216})-(\color{#D61F06}{180})(\color{#3D99F6}{24})+3(\color{#EC7300}{420})=\color{#69047E}{2124}

Calculate the sum:

cos A + cos B + cos C = 24 ( a b + a c + b c ) 3 a b c ( a 3 + b 3 + c 3 ) 2 a b c = 24 ( 180 ) 3 ( 420 ) 2124 2 ( 420 ) = 4320 1260 2124 840 = 936 840 = 39 35 \cos A + \cos B + \cos C\\ =\dfrac{24(\color{#D61F06}{ab+ac+bc})-3\color{#EC7300}{abc}-(\color{#69047E}{a^3+b^3+c^3})}{2\color{#EC7300}{abc}}\\ =\dfrac{24(\color{#D61F06}{180})-3(\color{#EC7300}{420})-\color{#69047E}{2124}}{2(\color{#EC7300}{420})}\\ =\dfrac{4320-1260-2124}{840}\\ =\dfrac{936}{840}\\ =\dfrac{39}{35}

p = 39 , q = 35 11 ( p + q ) + 25 = 11 ( 39 + 35 ) + 25 = 11 ( 74 ) + 25 = 814 + 25 = 839 \implies p=39,\;q=35\\ 11(p+q)+25\\ =11(39+35)+25\\ =11(74)+25\\ =814+25\\ =\boxed{839}

Wow colorful... (+1) :-)

Rishabh Jain - 4 years, 11 months ago

Log in to reply

The solutions I wrote for your questions are even more colorful

Hung Woei Neoh - 4 years, 11 months ago

Log in to reply

Lol .. Your each and every solution is colorful.. XD

Rishabh Jain - 4 years, 11 months ago

Log in to reply

@Rishabh Jain Well, all credits go to Mr. @Chew-Seong Cheong . I got the idea of using colors in my solution while reading some of his solutions

Hung Woei Neoh - 4 years, 11 months ago

0 pending reports

×

Problem Loading...

Note Loading...

Set Loading...